Bạn chưa đăng nhập. Vui lòng đăng nhập để hỏi bài

Những câu hỏi liên quan
Nguyễn Thị Huệ
Xem chi tiết
Lý Quốc Bảo
19 tháng 1 2016 lúc 19:17

1/ khi m=3 ta có

x+3y=3

3x+4y=7

<=>x=3-3y

      3(3-3y)+4y=7

<=>x=3-3y

      3y+4y=7

<=>x=3-3y

      7y=7

==>y=1

<=>x=3-3y

=>x=3-3.1

=>x=3-3

==>x=0

vây x=0     ; y=1

~Miêu Nhi~
Xem chi tiết
Ác Mộng Màn Đêm
Xem chi tiết
Nguyễn Lê Phước Thịnh
19 tháng 3 2021 lúc 20:14

a) Ta có: \(\left\{{}\begin{matrix}3x+y=3\\2x-y=7\end{matrix}\right.\)

\(\Leftrightarrow\left\{{}\begin{matrix}5x=10\\2x-y=7\end{matrix}\right.\Leftrightarrow\left\{{}\begin{matrix}x=2\\y=2x-7=2\cdot2-7=-3\end{matrix}\right.\)

Vậy: Hệ phương trình có nghiệm duy nhất là (x,y)=(2;-3)

Nguyễn Lê Phước Thịnh
19 tháng 3 2021 lúc 20:16

b) Ta có: \(7x^2-2x+3=0\)

a=7; b=-2; c=3

\(\Delta=\left(-2\right)^2-4\cdot7\cdot3=4-84=-80< 0\)

Suy ra: Phương trình vô nghiệm

Vậy: \(S=\varnothing\)

Pham Trong Bach
Xem chi tiết
Cao Minh Tâm
21 tháng 12 2019 lúc 4:09

Đáp án: C

Le Xuan Mai
Xem chi tiết
Nguyễn Lê Phước Thịnh
29 tháng 12 2023 lúc 21:55

Để hệ phương trình có nghiệm duy nhất thì \(\dfrac{m}{2}\ne\dfrac{2}{-4}=-\dfrac{1}{2}\)

=>\(m\ne-1\)

\(\left\{{}\begin{matrix}mx+2y=1\\2x-4y=3\end{matrix}\right.\)

=>\(\left\{{}\begin{matrix}2mx+4y=2\\2x-4y=3\end{matrix}\right.\Leftrightarrow\left\{{}\begin{matrix}x\left(2m+2\right)=5\\2x-4y=3\end{matrix}\right.\)

=>\(\left\{{}\begin{matrix}x=\dfrac{5}{2m+2}\\4y=2x-3=\dfrac{10}{2m+2}-3=\dfrac{10-6m-6}{2m+2}=\dfrac{-6m+4}{2m+2}\end{matrix}\right.\)

=>\(\left\{{}\begin{matrix}x=\dfrac{5}{2m+2}\\y=\dfrac{-6m+4}{8m+8}=\dfrac{-3m+2}{4m+4}\end{matrix}\right.\)

x-3y=7/2

=>\(\dfrac{5}{2m+2}-\dfrac{3\cdot\left(-3m+2\right)}{4m+4}=\dfrac{7}{2}\)

=>\(\dfrac{10+3\left(3m-2\right)}{4m+4}=\dfrac{7}{2}\)

=>\(\dfrac{10+9m-6}{4m+4}=\dfrac{7}{2}\)

=>\(\dfrac{9m+4}{4m+4}=\dfrac{7}{2}\)

=>7(4m+4)=2(9m+4)

=>28m+28=18m+8

=>10m=-20

=>m=-2(nhận)

Pham Trong Bach
Xem chi tiết
Cao Minh Tâm
16 tháng 2 2017 lúc 16:50

2 x + 3 y = − 2 3 x − 2 y = − 3 ⇔ 4 x + 6 y = − 4 9 x − 6 y = − 9 ⇔ 13 x = − 13 2 x + 3 y = − 2 ⇔ x = − 1 y = 0

Vậy hệ đã cho có nghiệm duy nhất (x; y) = (−1; 0)

 x – y = −1 – 0 = −1

Đáp án: A

Nguyễn Quỳnh Nga
Xem chi tiết
Nguyễn Quỳnh Nga
29 tháng 6 2017 lúc 19:32

giúp mk với mk cần gấp

Thiên An
30 tháng 6 2017 lúc 17:04

Ta có định lý sau:

Hệ  \(\hept{\begin{cases}a_1x+b_1y=c_1\\a_2x+b_2y=c_2\end{cases}}\)  

- Có 1 nghiệm duy nhất khi  \(\frac{a_1}{a_2}\ne\frac{b_1}{b_2}\)

- Có vô số nghiệm khi  \(\frac{a_1}{a_2}=\frac{b_1}{b_2}=\frac{c_1}{c_2}\)

Do đó  \(\hept{\begin{cases}2x+y=5\\mx-y=-7\end{cases}}\)   có 1 nghiệm duy nhất  \(\Leftrightarrow\)  \(\frac{2}{m}\ne\frac{1}{-1}\)  \(\Leftrightarrow\)  \(m\ne-2\)

Hệ pt ko thể có vô số nghiệm vì  \(\frac{1}{-1}\ne\frac{5}{-7}\)

Quỳnh Anh
Xem chi tiết
Hồng Phúc
15 tháng 12 2020 lúc 20:32

Đặt \(x+\dfrac{1}{x}=a;y+\dfrac{1}{y}=b\left(\left|a\right|\ge2;\left|b\right|\ge2\right)\)

\(\left\{{}\begin{matrix}x+\dfrac{1}{x}+y+\dfrac{1}{y}=5\\x^3+y^3+\dfrac{1}{x^3}+\dfrac{1}{y^3}=15m-25\end{matrix}\right.\)

\(\Leftrightarrow\left\{{}\begin{matrix}x+\dfrac{1}{x}+y+\dfrac{1}{y}=5\\\left(x^3+\dfrac{1}{x^3}\right)+\left(y^3+\dfrac{1}{y^3}\right)=15m-25\end{matrix}\right.\)

\(\Leftrightarrow\left\{{}\begin{matrix}x+\dfrac{1}{x}+y+\dfrac{1}{y}=5\\\left(x+\dfrac{1}{x}\right)^3-3\left(x+\dfrac{1}{x}\right)+\left(y+\dfrac{1}{y}\right)^3-3\left(y+\dfrac{1}{y}\right)=15m-25\end{matrix}\right.\)

\(\Leftrightarrow\left\{{}\begin{matrix}x+\dfrac{1}{x}+y+\dfrac{1}{y}=5\\\left(x+\dfrac{1}{x}\right)^3+\left(y+\dfrac{1}{y}\right)^3-3\left(x+\dfrac{1}{x}+y+\dfrac{1}{y}\right)=15m-25\end{matrix}\right.\)

\(\Leftrightarrow\left\{{}\begin{matrix}x+\dfrac{1}{x}+y+\dfrac{1}{y}=5\\\left(x+\dfrac{1}{x}\right)^3+\left(y+\dfrac{1}{y}\right)^3=15m-10\end{matrix}\right.\)

\(\Leftrightarrow\left\{{}\begin{matrix}a+b=5\\a^3+b^3=15m-10\end{matrix}\right.\)

\(\Leftrightarrow\left\{{}\begin{matrix}a+b=5\\\left(a+b\right)^3-3ab\left(a+b\right)=15m-10\end{matrix}\right.\)

\(\Leftrightarrow\left\{{}\begin{matrix}a+b=5\\125-15ab=15m-10\end{matrix}\right.\)

\(\Leftrightarrow\left\{{}\begin{matrix}a+b=5\\ab=9-m\end{matrix}\right.\)

\(\Rightarrow a,b\) là nghiệm của phương trình \(t^2-5t+9-m=0\left(1\right)\)

a, Nếu \(m=3\), phương trình \(\left(1\right)\) trở thành

\(t^2-5t+6=0\)

\(\Leftrightarrow\left[{}\begin{matrix}t=2\\t=3\end{matrix}\right.\)

\(\Leftrightarrow\left[{}\begin{matrix}\left\{{}\begin{matrix}a=2\\b=3\end{matrix}\right.\\\left\{{}\begin{matrix}a=3\\b=2\end{matrix}\right.\end{matrix}\right.\)

TH1: \(\left\{{}\begin{matrix}x+\dfrac{1}{x}=2\\y+\dfrac{1}{y}=3\end{matrix}\right.\Leftrightarrow\left\{{}\begin{matrix}\left(x-1\right)^2=0\\y^2-3y+1=0\end{matrix}\right.\Leftrightarrow\left\{{}\begin{matrix}x=1\\y=\dfrac{3\pm\sqrt{5}}{2}\end{matrix}\right.\)

TH2: \(\left\{{}\begin{matrix}x+\dfrac{1}{x}=3\\y+\dfrac{1}{y}=2\end{matrix}\right.\Leftrightarrow\left\{{}\begin{matrix}x=\dfrac{3\pm\sqrt{5}}{2}\\y=1\end{matrix}\right.\)

Vậy ...

b, \(\left(1\right)\Leftrightarrow t=\dfrac{5\pm\sqrt{4m-11}}{2}\left(m\ge\dfrac{11}{4}\right)\)

\(\left(1\right)\Leftrightarrow\left\{{}\begin{matrix}a=\dfrac{5\pm\sqrt{4m-11}}{2}\\b=\dfrac{5\mp\sqrt{4m-11}}{2}\end{matrix}\right.\)

\(\Leftrightarrow\left\{{}\begin{matrix}x+\dfrac{1}{x}=\dfrac{5\pm\sqrt{4m-11}}{2}\\y+\dfrac{1}{y}=\dfrac{5\mp\sqrt{4m-11}}{2}\end{matrix}\right.\)

\(\Leftrightarrow\left\{{}\begin{matrix}2x^2-\left(5\pm\sqrt{4m-11}\right)+2=0\left(2\right)\\2y^2-\left(5\mp\sqrt{4m-11}\right)+2=0\end{matrix}\right.\)

Yêu cầu bài toán thỏa mãn khi phương trình \(\left(2\right)\) có nghiệm dương

\(\Leftrightarrow\left\{{}\begin{matrix}\Delta=\left(5\pm\sqrt{4m-11}\right)^2-16\ge0\\\dfrac{5\pm\sqrt{4m-11}}{2}>0\\1>0\end{matrix}\right.\)

\(\Leftrightarrow...\)

Hồ Minh Khang
Xem chi tiết
Nguyễn Hoàng Minh
1 tháng 12 2021 lúc 17:26

\(a,\text{Thay }x=-2;y=3\\ HPT\Leftrightarrow\left\{{}\begin{matrix}3m-2=4\\3-2n=-3\end{matrix}\right.\Leftrightarrow\left\{{}\begin{matrix}m=2\\n=3\end{matrix}\right.\\ b,HPT\Leftrightarrow\left\{{}\begin{matrix}x=4-my\\n\left(4-my\right)+y=-3\end{matrix}\right.\Leftrightarrow\left\{{}\begin{matrix}x=4-my\\4n-mny+y=-3\end{matrix}\right.\\ \Leftrightarrow\left\{{}\begin{matrix}x=4-my\\y\left(mn-1\right)=4n+3\end{matrix}\right.\)

HPT có vô số nghiệm \(\Leftrightarrow\left\{{}\begin{matrix}mn-1=0\\4n+3=0\end{matrix}\right.\Leftrightarrow\left\{{}\begin{matrix}m=-\dfrac{4}{3}\\n=-\dfrac{3}{4}\end{matrix}\right.\)

Nguyễn Thị Huệ
Xem chi tiết